LSAT and Law School Admissions Forum

Get expert LSAT preparation and law school admissions advice from PowerScore Test Preparation.

 Administrator
PowerScore Staff
  • PowerScore Staff
  • Posts: 8919
  • Joined: Feb 02, 2011
|
#84802
Complete Question Explanation

Weaken, Except - CE. The correct answer choice is (B).

Answer choice (A):

Answer choice (B): This is the correct answer choice.

Answer choice (C):

Answer choice (D):

Answer choice (E):

This explanation is still in progress. Please post any questions below!
 cgleeson
  • Posts: 75
  • Joined: Feb 13, 2022
|
#94620
Hi,
I was stuck between C & D... answer choice B seemed to me as though it didn't weaken EXCEPT just offered another reason. In any event I wasn't sold. D seemed totally irrelevant and I ended up with that answer because it didn't seem to do anything to the question. Where may have I gone wrong?
Many thanks,
Chris 8-)
 Rachael Wilkenfeld
PowerScore Staff
  • PowerScore Staff
  • Posts: 1358
  • Joined: Dec 15, 2011
|
#94677
Hi Chris,

Our conclusion here is that the glasses are worn because of the tendency towards hypochondria and depression. So the cause is hypochondria/depression, and the effect is the tinted glasses. To weaken that, we are looking for answer choices that provide an alternate cause, show the cause occurs without the effect, show the effect occurs without the cause, that the cause and effect are reversed, or that there was a problem with the underlying data. Our incorrect answers will all weaken, while the correct answer choice will either strengthen or have no impact.

Let's look at answer choices (B), (C), and (D).

Answer choice (B): This is our correct answer choice. It has no impact on the causal relationship because we don't know or care if how the doctor should respond to the medical complaints because this would be after the causal chain occurs. The doctor's response, or lack thereof, to medical complaints by tinted glasses wearers does not impact if doctors should investigate the claims of those patients.

Answer choice (C): This answer choice weakens the underlying data. Since the conclusion is about people in Britain, if the data was regarding a different environment, that would weaken the conclusion.

Answer choice (D): This weakens the conclusion by providing an alternate cause for the tinted glasses wearing, namely fashion.

Hope that helps!

Answer choice (C):

Get the most out of your LSAT Prep Plus subscription.

Analyze and track your performance with our Testing and Analytics Package.